The smart way to learn. The smart way to teach.

MRCOG PART 2 SBAs and EMQs

Course PAID
notes336
EMQ1502
SBA2115
Do you realy want to delete this discussion?
Forum >>

Essay 346

Janaki Posted by Janaki veer  D.
Information is obtained from her history about the nature of the pain, radiation, aggravating and relieving factors. Adequate time is allotted for her to speak about her complaints and careful listening is ensured. She should be asked about the effect of the symptoms on her quality of life.she should be asked sensitively about the social and psychological pressures at work or at home. Enquiry is made regarding sexual or physical abuse. Menstrual history is taken which includes regularity, cycle length, flow and LMP. Gastrointestinal causes can also present in cyclical manner. Loss of weight, bloody stools and passage of mucus points towards inflammatory bowel disease. She should be asked about change of bowel habit ( change in frequency and form), pain relieved by defaecation suggesting irritable bowel syndrome. If she is a parous woman, history is taken about the previous pregnancies, their outcomes including caesarean delivery and terminatiions. She should be asked about the details of previous and present methods of contraception. Her wishes for future fertility are explored. Information is obtained about the treatment details, their effect on her symptoms, and problems encountered along with her expectations. History is taken about number of sexual partners, recent change of partner in the last 6 months, previous sexually transmitted infections and treatment taken. Abdominal examination is performed to detect tender areas and  obvious abdominal masses. If she is not sexually active, pelvic examination is avoided. If she is sexually active, speculum and vaginal examination are undertaken to note for purulent vaginal discharge, uterine size, position, fixity and adnexal masses. B) the gold standard investigation for diagnosis of endometriosis is laparoscopy. With prior knowledge of various appearances of endometriosis, laparoscopy with two probes is done to visualize  pouch of Douglas, uterosacral ligaments, tubes and ovaries. Extent of endometriosis is assessed using American Fertility Society (AFS) classification and scoring is done. A score of 6-15 makes the diagnosis of mild endometriosis and 16-30 score enables diagnosis of severe endometriosis . Endometriomas more than 3 cm and suspicious areas of peritoneum and adhesions are biopsies to confirm the diagnosis. C) Treatment is directed towards the symptom relief taking account of her fertility wishes or her request about contraception. All medical therapies have similar efficacy but varied side effect profile. COCP is common first line therapy given in cyclical or tricyclical fashion or given continuously. COCP is effective in achieving pain relief apart from provision of contraception if she doesn't want to conceive.  Progestogens in sufficient doses to induce amenorrhoea  ( medroxy progesterone acetate, DMPA) are effective in pain . Side-effects includeweight gain, mood change / depression, acne, bloating; prolonged use may lead to hypo-oestrogenaemia and loss of bone mineral density. DMPA use is associated with risk of irregular bleeding. Danazol is androgenic anti-oestrogen and anti-progestogen; also has direct inhibitory effects on endometrium. It is Effective therapy but limited by side-effects such as acne, hirsutism, muscle aches, weight gain,  hot flushes, breast atrophy which are reversible. Deepening of the voice may be irreversible. She is advised to use barrier contraception as risk of virilisation of female fetus. Gestrinone is 19-nortestosterone derivative, androgenic anti-oestrogen and anti-progestogen which is effective in symptom releif with milder side effects compared to Danazol. Requires barrier contraception as it can virilise female fetus. GnRH agonists are associated with menopausal side-effects and treatment should be limited to 6 months as there is loss of 6% of bone mineral density after a 6 months course. Bone loss may not be entirely reversible.  Add-back therapy with tibolone, a progestogen or a low dose oestrogen may allow GnRH agonists to be used for up to 2 years without significant loss of bone density. LNG IUS is effective in reducing endometriosis related pain with effect maintained for up to 3 years The aims of surgical treatment are to restore normal anatomy, excise disease and preserve normal tissue. Laparoscopic ablation of moderate endometriosis is effective in relieving pain at 6 months follow-up. Excisional surgery was associated with significant symptomatic improvement at 6 months.  The effect of excision of moderate - severe disease with adhesiolysis on pregnancy rates remains uncertain. If she wishes to conceive IVF - embryo transfer is effective treatment for endometriosis associated infertility. Laparoscopic ovarian cystectomy is recommended for endometriomas ≥ 4 cm in diameter prior to IVF. Laparoscopy is associated with risks of visceral injury (bowel and bladder) and blood vessel injury.
Posted by Yingjian C.

a) My initial assessment would be to take a menstrual history especially the timing of the pelvic pain in relation to her periods, the flow, if she has regular periods and how these symptoms affect her quality of life.  Her desires for fertility and contraception, details of previously tried medications/ treatment options would be sought as well. Any urinary or bowel symptoms, such as pain or bleeding would be asked as it may suggest deeply infiltrating endometriosis. Her goals of seeking treatment would be asked as this would affect her treatment options.

 

Next a physical examination would be carried out looking out for any abdominal tenderness or pelvic masses. A vaginal examination would be performed looking out for uterine size, mobiliy, tenderness, adnexa masses, cervical excitation and a rectal examination to examine the uterosacral ligaments and pouch of douglas.

 

Lastly a transvaginal ultrasound would be useful in detecting adnexa masses. A magnetic resonance imaging scan, transrectal ultrasound scan or intravenous pyelogram may be useful if the history or physical examination suggests deeply infiltrating endometriosis.    

 

b) Laparoscopy is the gold standard for making a diagnosis of endometriosis. The American Fertility Scoring System can be used to grade the severity of the disease, which may not correlate with the severity of her symptoms. In this scoring system, the peritoneal adhesions, pouch of Douglas, ovaries, ovarian fossae, fallopian tubes are surveyed and points assigned accordingly and added together to give the severity of the endometriosis.

 

c) Her treatment options would be medical or surgical. Medical treatment options include simple analgesia such as non steroidal anti inflammatory drugs. This option may be effective in some patients but side effects such as gastric ulceration and its anti ovulatory effect in the mid cycle must be considered. Hormonal methods of treatment include combined oral contraceptive pills which are effective in treating menstrual symptoms in women who desire contraception at the same time. It can be taken in the conventional manner, continuously or tricycling. Progestrogens in doses which induce amenorrhea, such as depo provera can be tried for pain relief. However side effects like irregular bleeding, weight gain, mood changes, acne, bloating and osteoporosis with long term use must be considered. Danazol which is an androgenic antiestrogen and antiprogestrogen can be tried, however its use is limited by its side effect profile which includes acne, hirsutism, weight gain, oily skin, breast atrophy and voice deepening. It should not be used in a patient who desires fertility as it can caue virilisation of a female fetus, and its use should be limited to a duration of six months. Gestrinone – an androgenic antiestrogen and antiprogestrogen drug which is a 19-nortestosterone derivative is associated with similar side effects although milder than danazol. Similarly it is not suitable for use in a patient who desires fertility. Gonadotrophin releasing hormone agonists can be used as well however this is associated with menopausal side effects and treatment duration limited to 6 months due to risk of bone loss. Add back therapy may allow for a longer duration of use. Levonorgestrel intrauterine system is also effective in reducing endometriosis related pain.

 

Surgical options of treatment would include ablation of endometriosis, adhesiolysis and excision of endometriomas during laparoscopy. This is known to cause symptomatic improvement however results may not be sustained, and the risks bowel/ bladder injury during surgery is increased especially if there are extensive adhesions. The other surgical option is pre sacral neurectomy for pain relief however this is a complex surgery. Laparoscopic uterine nerve ablation does not help to relief pain associated with endometriosis. As a last resort pelvic clearance with post operative hormone replacement therapy can be considered. 

Ali Naveed Haq Posted by Ali Naveed Haq H.

 

A healthy 35 year old woman is referred to the gynaecology clinic because of a 12 months history of pelvic pain, painful periods and pain during intercourse. Her symptoms have not responded to simple analgesia. (a) Discuss your clinical assessment [6 marks]. (b) She is found to have moderately severe endometriosis. Discuss the criteria for making this diagnosis [4 marks]. (c) Discuss and justify her treatment options [10 marks]

 

 

a) I will take a detailed history of the impact of this pain on the quality of her , does this pain lead to missing work, the timing with menstrual cycle is it from the start or late which may signify adenomyosis, presence of dyschazia which may point to bowel endometriosis, her desire for fertility , including parity , any treatment taken in that context . Associated hemoptysis pointing to lung involvement, type of dysparunea superficial or deep. Associated vaginal discharge , intermenstrual bleeding and post coital bleeding suggesting infective origin. History of sexual abuse , poor relationship with the partner may point towards psychological etiology, history of trauma to the pelvis.

On examination I will look , spinal , problems , take appropriate swabs if there is history of discharge and if there is none then pelvic examination for tenderness , fixed retroversion, thickened uterosacral ligaments, endometriotic deposits may rarely be noted in the vagina , rarely on incisional sites . The pelvic examination is best done for this purpose in the menstrual phase.

Ultrasound for pelvic mass , cyst size and version of the uterus as in the presence of adhesions , fixed retroversion etc. transvaginal scan is helpful . Ca 125 is raised in endometriosis but highly nonspecific. Diagnostic laparoscopy is gold standard in detecting the problem this will depend upon the debility suffered by the lady if this pain has a significant impact on her life than laparoscopy may  be undertaken , after explanation of underlying risks of associated bowel and vessel damage. In addition consent of therapeutic intervention may also be under taken.

 

b) Endometriosis is diagnosed and classified into minimal , mild, moderate and severe disease according to revised American fertility society criteria. This prognostic for providing the severity of disease according to location, size, extent, endometriotic implants and adhesions whether filmy or dense of , however does not predict future conception or treatment benefit.

The score is 1-5 for minimal disease, 6-15 mild , 16-40 moderate and > 40 severe disease. Endometriotic choclate cysts and deep infiltration with dense adhesions signify severe disease.

 

c) The treatment depends upon her fertility desire. As medical therapy may be of help in relieving pain but a total waste of precious time when fertility is required.

The treatment options for pain relief include use of combined oral contraceptive pills , Danazole , progestogens , medroxy progesterone acetate. These provide benefit in terms of symptoms but side effects of danazole may be unacceptable to some women. Combined oral contraceptive pill is effective when used for long term but may not be suitable for all women. GnRh analogues are highly effective but can be used only for 3- 6 months these are expensive, produce osteoporosis and menopausal symptoms, in addition the drawback with medical therapy is that symptoms return upon disuse.

 

Surgical options like laparscopic adhesiolysis , ablasion of endometriotic implants , removal of cysts are useful and have been found to provide relief of sypmptoms and is the recommended treatment. It is also highly beneficial in women planning pregnancy. Where IUI , or IVF may be undertaken improving success of treatment .

Laparotomy for clearing endometriosis can also be performed , there is no use of using postoperative hormone therapy in places where laparoscopic expertise is not available.

In cases of severe pain total abdominal hysterectomy with removal of ovaries and pelvic implants may provide relief. This can be backed up with using HRT.

 

Accupuncture , TENS and alternative therapy may be useful for some ladies.

 

 

  

ans to essay 346 Posted by SARO K.

 

Ans to essay 346:

(a)I will get a detailed history about her impact of symptoms on quality of life,menstrual history-LMP,any history of increased flow,obstetric history-prior pregnancies and the outcome , future reproductive options,bowel symptoms-constipation ,diarrhea,pain while passing stools ,b;eeding PR ,loss of weight,passing mucus  pointing towards Inflammatory/irritable bowell disease. Ask about  urinary symptoms like  hematuria to assess her urinary tract involvement.I will ask about contraceptive history-previously what was she using , her concerns,  difficulties if any and improvement of symptoms. I will ask about sexual  history regarding frequency of intercourse,degree of dyspareunia and severe fatigue following .Prior sexually transmitted disease and its treatment.I will ask abt previous surgery  to rule out adhesion.

I will examine her BMI,Bp,per abdomen to look for lower abdomen tenderness,mass ,per speculum for cervix , any vaginal lesion(bluish spots) and swabs to rule out infection and per vaginal examination to look for size and position of uterus,adnexal mass(chocolate cyst) forniceal  tenderness,mobility of uterus,nodules in the posterior fornix,thickening of uterosacral ligament  .Per rectal examination to look for thickening of rectovaginal septum and nodularity

(b)Scoring depends on the  size of the lesion(less than 1cm-1,1-3 cm- 2 ,more than 3 cm -4 and density of adhesion in the peritoneum,tube,ovary,POD obliteration (partial 4,complete 40)and fimbria enclosed(16).stage I  1-5 ,Stage II 6-15,stage III16-40 and stage IV more than 40.Scoring does not correlate with pain /infertility.

©Treatment depends on her reproductive plans and contraceptive options.If  she is not  keen on conception and concerned about symptom relief with need of contraception medical management  is offered.Choice depends on her  side effect profile as they are all equally effective.

COMBINED ORAL CONTRACEPTIVE PILL can be used cyclically/continuously/tricyclically for suppression of ovulation will relieve her pain symptoms  as well serve the purpose if contraception.PROGESTOGENS can be given  to suppress menstruation and hence symptom relief.I t can be given in any form pill/injectable/IUD depending on the compliance and contraceptive options.Depo provera  and IUD  serves  as LARC with good improvement of symptoms with amennohea (LNG IUD)and irregular bleeding ( injectables).Danazol  is androgenic ,antiestrogenic and antiprogestogenic  with good pain relief.Side effects include acne,mood changes,wtgain,voice change(irreversible),breast atrophy  and  benign hepatic adenoma.Contraception should be used  to prevent effects on fetus. Gestrinione  is 19 nor testosterone derivative  with anti estrogenic,antiprogestogenic and androgenic  effects with biweekly administration.GnRh agonists causes pseudomenopausal state with relief of symptoms  and loss of BMD(6percent in 6 months) ,hotflushes.Can be used with add back therapy with tibilone/progestogen /low dose estrogen  for more than 6 months  without loss of BMD.Symptom recurrence is common following  stoppage of therapy. Surgical  management includes excision ,adhesiolysis , laser ablation/coagulation.Laparoscopc uterine nerve ablation has no relief of symptoms  with uterine prolapse later. Presacral neurectomy  cause symptom relief with high morbidity.If  SYMPTOMS not responding to treatment,TAH with BSO can be offered  as final resort along  with HRT.

                If she want to conceive, medical management suppresses ovulation and is not THE option.Laproscopic excision of lesion  can be offered with improvement in fertility compared to ablation/coagulation with inherent  problems like injury to bowel bladder and blood vessel.IVF can be offered with GnRh agonists 3 months prior,will improve pregnancy rate.

 

 

 

 

 

Posted by Muthu M.

A.  I like more detailed history to find out the pelvic pain whether it is constant or more during her periods, whether she suffers from ovulation pain, the deep dyspareunia and dyschezia, haematuria and bleeding per rectum are some of the specific history towards diagnosing endometriosis.  I would like to ask about bowel habit to rule out irritable bowel syndrome; about pain during passing urine, urgency to rule out interstitial cystitis; previous pelvic infection and use of condoms to rule out chronic pelvic infection.

I will examine the patient, in abdominal examination to look for mass in the lower abdomen arising from pelvis; check for endometriomas, and any specific area of tenderness.  On speculum examination, I will do swabs to rule out pelvic infection, vaginal examination: the uterus size, mobility (fixed due to scarring), adnexal masses (endometiomas) and tenderness, cervical excitation (pelvic infection), and nodularity at pouch of douglas, per rectal examination if bleeding per rectum – to diagnose signs of endometriosis or chronic pelvic infection.

To diagnose the endometriosis, she needs laparoscopy to look for endometriosis spots or other signs of endometriosis at pouch of douglas, ovarian fossa, ovaries and fallopian tubes to be checked.

 

B. It may be a clinical diagnosis - if anyone with above symptoms and signs on vaginal examination, we could give combined contraceptive pills and if their symptoms improve, the diagnosis is endometriosis.  The CA125 may be raised, but it is non-specific.  However, the gold standard for diagnosis is ‘Laparoscopy’.  By laparoscopy, one could see the powdered lesion, or extensive scarring, distortion of tubes due to endometriosis.  It depends on size of lesion and involvement of structures; it will be scored individually as 1-4.  The areas of involvement are pouch of douglas, ovarian fossa, ovaries and fallopian tubes, and also endometriomas involving ovaries.  The peritoneum should be checked for endometriosis spots.  The addition of those values will give us the numbers, which will help to do staging of endometriosis like mild, moderate and severe- there are 4stages according to endometriosis society guideline/ staging. 

 

C.  The treatment depends on how it affects her life: quality of life, fertility and effect on family life and personal relationship.  The treatment can be medical and surgical; medical is non-hormonal and hormonal.  Non-Hormonal is like NSAIDs.

Hormonal can be given provided if she does not have any plans to try for pregnancy; combined contraceptive pills – it can be given as tricyclic to reduce number of bleeding cycle per year. Could have progesterone only pills, depo injection – it has its side effects like weight gain, irregular bleed and mood swings.  She could try danazol which is androgenic anti oestrogen.  She needs additional barrier method of contraception to avoid virulization of female fetus if she falls pregnant while on Danazol.  She could try, GnRH analogue to stop the pituitary ovarian function for 6 months.  She could have it longer, if she could have additional add back Hormone replacement therapy. 

Next option is surgical management; simple ablation to extensive resection.  Patient should be counseled regarding uretric injury and bowel injury.  If patient is not planning to have any pregnancies in life, she could be offered extensive resection like hysterectomy, bilateral salpingo-oophorectomy, resection of involved bowel, part of bladder involvement.

If she would like to become pregnant, she could ablation and resection of endometriosis and also adhensiolyis of distorted tubes.  The chances of pregnancy are good after resection of endometriosis or 6 months of GnRH and then try for pregnancy.

The treatment must be individualized and also needs counseling and psychological assessment and psychological support.

essay 346 Posted by vaneeza K.

A.Detail history will be asked to assess severity of symptoms and their effect on quality of life.Non cyclical ,chronic pain worsened by certain body movements and postures,trigerred by ovulution,bowel movements,urination and mensaturation are due to adhesions so history of previuos surgeries should be asked. Pain started before periods,becomes spasmodic  with mensaturation ,continue through out periods,lessen at end is typically due to endometriosis. Pain starts before periods and relieved with mensaturation is due to primary dymenorrhoea. History of difficult defecation, mucous and blood in stool or rectal bleeding should be asked as these are due to inflammatory bowel disease.Dysuria and haematuria indicate bladder involvement. Superficial ,burning pain during intercourse is due to pelvic infection while deep dysparunea that continues as dull ache after intercourse indicate endometriosis,should be asked.  Mensatural history including cycle length,regularity heavy flow?intermensatural bleeding and premensatural spotting is to be asked. She will be asked about previous obstetrical history,difficulty in conceving,previous and current contraception use and future fertility intentions to decide about treatment. History of no of sexual partners,change of partners during last 6 months,use of condoms.and previous STI must be enquried as her three complains may be due to PID.I will examine abdomen for scars and palpable mass.Vaginal examination for uterine size,tenderness,mobility,adenaxal mass , tenderness and nodularity in pouch of douglas and cervical excitation will be done.On speculum exam bluish spots may be visible and swabs can be taken.If there is rectal bleeding history ,than rectal exam to be performed at same time with vaginal examination.

B.All classifications of endometriosis are subjective and correlated poorly with pain. American Fertility Society scoring system has divided endometriosis into mild,moderate,sever and extensive disease depending upon size of leision ,density of adhesions in peritonium.ovaries and fallopian tubes.Its mild when scores (1-5) , moderate(6-15) , severe (16-30) and extensive (31-54).

 

C.Patient has option of medical, hormonal treatment but it will not improve fertility and risk of reccurence.COCP can be given in tricycling pattern, will improve pain symptoms ,provide contraception at same time if she wants.Progestogens medoxyprogesterone acetate,dydrogesterone and norethisterone can be used orally in sufficent  doses to induce amenorrhea,help in relieving pain. these are associated with bloatedness,nausea.fluid retention, mood changes depression ,breast tenderness and irregular bleeding.The LNG-IUS reduces endometriosis associated pain with symptom controll over 3 yrs.Danazol androgenic,antiestrogenic and anti progestrogenic ,is highly effective in acheving symptomatic relief in 90% of cases when given daily oral dose of 200mg Symptom will relapse with in average 6.1months after stoping treatment and can be given maximum for 6 months.It is associated with side effects like weight gain,acene, hirsutism,mood changes and mensatural irregularities that are reversible but voice changes are irreversible and treatment has to be stopped.it also causes dislipidaemia,liver adenoma and arterial thrombosis.Woman will be advised to use contraception as it causes virilization of female foetus.Gestrinone given twice weekly  as effective as danazol with less side effects ,also causes virilization of female foetus.

GnRH analogues improve 80% of symptoms with in 4 wks,improve signs (pelvic tenderness,&induration)and acheving >50% reduction in size of endrometriosis as determined by laparoscopy.It has initial flare up effect last for 1-2 wks associated with irregular vaginal bleeding followed by amennorhea .Side effects are vaginal dryness night sweats,hot flushes and 6% BMD loss with in 6 months that can be reduced by add back therapy with tibolone.

Surgery is another effective option with diseade recurrence rate at five yr follow up is 20%.Laparoscopic ablation for minimal to moderate disease effective in pain relief for 6 months in 60%  of patients and with  excision in about 80% of patients.Excision and ablation or cystectomy for ovarian endometrioma.For more extensive surgery MDT including colorectal and urology surgeon should be involved.

Complimentry therapies like TENS ,accupuncture can be offered ,infformation about support group should be provided.

ESSAY 346 Posted by Gulshan K.
  1. I will ask about severity of her symptoms and their effects on quality of life. I will ask about her menstrual history .Enquire about gastrointestinal symptoms like change in bowel habit, alternating constipation and diarrhea, rectal bleeding and weight loss will show gastrointestinal cause or its involvement in disease. Involvment of Urinary symptoms like bladder pain. Past history of PID and history of subfertility. Obstetrics history. Sexual abuse history. Enquire about psychological symptoms. Clinical examination of abdomen for tenderness, rebound tenderness, abdominal mass and distention. Pelvic examination to assess uterine size, mobility, its position, tenderness, adnexal tenderness, cervical tenderness and presence of nodules in   rectovaginal septum. Per rectal examination in case of rectal bleeding.
  2. Criteria include Signs and symptoms of endometriosis but they are nonspecific. Serum marker of CA 125 level do not provide adequate diagnostic accuracy.TVS is useful for diagnosis of endometrioma and normal finding may reassure woman  but it cannot diagnose peritoneal endometriosis. MRI may be useful in diagnosis of deeply infiltrating endometriosis. Diagnostic laparoscopy for visual inspection of pelvic organs and positive histology confirms the diagnosis but negative histology does not exclude endometriosis.
  3. Treatment options depend upon woman wishes and her concerns about symptoms control or subfertility. If woman want symptoms control and infertility is not an issue than she should provide hormonal treatment. For symptoms control. Use of COCP, progestogens, danazole and GnRH agonist all are equally effective in reliving symptoms by inhibiting ovulation but with different side effects profile. Preoperative assessment of disease extent and when clinical evidence of deeply infiltrating endometriosis, ureteric, bladder and bowel involvement should be assessed. Consider MRI, TVS IVP and barium enema studies to map disease. Surgical option consider if no response to medical treatment. Practice is to diagnose and ablation of all lesions for pain relief by laproscopy. Laparoscopic uterine nerve ablation does not relieve pain by itself and associated with uterine prolapsed and urinary problems. There is inconclusive evidence of presacral neurectomy. No role of preoperative and postoperative hormonal treatments. There is no role of complementary therapies in relieving endometriosis pain. Treatment should individualize according to patient need and wishes. If woman concerns is infertility than both partners should be evaluated even in presence of obvious cause. She needs treatment as there is no role of expectant management in moderate to severe endometriosis with infertility. There is no evidence of surgical treatment in improving infertility, but aim of surgery is to correct pelvic anatomy  to improve ease of monitoring and easier access to ovarian follicle for aspiration .In view of this laproscopy should arranged for ablation of visible lesions and restore pelvic anatomy. Endometrioma more than 4cm should be excised instead of ablation. Tubal flushing appears to improve pregnancy rate. IVF is appropriate treatment and met analysis showed down regulation with GnRH analogue for 3 months before IVF can improve live birth rate.                                                                
Posted by BHAWANA  P.

History need to be elicited about nature of pain cyclical in relation to her menstrual cycle or constant . severity of pain need to be assessed- how it is affecting her quality of life. Also i will ask about dyspareunia whether it is superficial or deep and whether or not she is able to complete the act( severity).I will enquire about her LMP,  periods- cycle, duration, amount of flow and dysmenorrhoea. I will also ask about intermenstrual or post-coital bleeding ( PID).sexual history need to be elicited and any history of multiple sexual partners.I will also ask about her parity and her future wishes of fertility.Is she on any contraception- Copper IUD (dysmennorhoea) or OCPs, mirena coil ( beneficial in her pain).

I will ask about bladder symptoms- pain during filling and relief on passing urine,urgency, frequency ( interstitial cystitis),Bowel symptoms- passage of mucus,change in frequency /form of stools(irritable bowel syndrome). I will elicit history of any operations in the past( intra-abdominal adhesions).

On abdominal examination- i will look for any tenderness in abdomen-site and rebound tenderness( rule out any acute cause of pain).on vaginal examination- assess any cervical tenderness, adenexal tenderness and tenderness in posterior fornix over uterosacral ligaments.

B)endometriosis is clinical diagnosis- with classical featues being deep dyspareunia, dyschezia and cyclical pain.A 3 months course of combined pills or GnRH analogues can be given to see whether it supresses pain or not which can be helpful in making diagnosis.

Laparoscopy is gold standard to make diagnosis and assessment need to be done according to the site involved and size of endometriomas.

Ultrasound can be done to determine size of endometriomas to plan surgery and rule out other pelvic pathology but not routinely performed to diagnose endometriosis.

c)Treatment options are directed according to patient wishes of fertility and severity of symptoms.Simple analgesia like NSAIDS can be given to releive pain. Oral contraceptive pills can be given for ovarian suppression releiving pain due to endometriosis.Progesterones like DMPA, mirena can be used to releive pain .

GnRH analogues with add back therapy like tibolone can be given again for ovarian suppression and relief of pain. Anti-estrogens like danazol can be used but they are not very popular because of their side-effect profile.

In mild to moderate endometriosis - endometriosis ablation can be done to relieve pain but there is not much evidence that it improves fertility. In severe endometriosis- excision of endometriosis and ovarian endometriomas (>5cm)has shown to be effective to improve fertility but there are no RCTs  done so far.Colorectal surgeons may need to be invovled according to the extent of involvement of endometriosis.The severity of endometriosis co-relates poorly with symptoms so detailed counselling of the patient need to be done before embarking on major surgery.

As a last resort, Pelvic clearance can be done after discussion with the patient and may need to continue on HRT after operation.

ESSAY 346 Posted by Gulshan K.
  1. I will ask about severity of her symptoms and their effects on quality of life. I will ask about her menstrual history .Enquire about gastrointestinal symptoms like change in bowel habit, alternating constipation and diarrhea, rectal bleeding and weight loss will show gastrointestinal cause or its involvement in disease. Involvment of Urinary symptoms like bladder pain. Past history of PID and history of subfertility. Obstetrics history. Sexual abuse history. Enquire about psychological symptoms. Clinical examination of abdomen for tenderness, rebound tenderness, abdominal mass and distention. Pelvic examination to assess uterine size, mobility, its position, tenderness, adnexal tenderness, cervical tenderness and presence of nodules in   rectovaginal septum. Per rectal examination in case of rectal bleeding.
  2. Criteria include Signs and symptoms of endometriosis but they are nonspecific. Serum marker of CA 125 level do not provide adequate diagnostic accuracy.TVS is useful for diagnosis of endometrioma and normal finding may reassure woman  but it cannot diagnose peritoneal endometriosis. MRI may be useful in diagnosis of deeply infiltrating endometriosis. Diagnostic laparoscopy for visual inspection of pelvic organs and positive histology confirms the diagnosis but negative histology does not exclude endometriosis.
  3. Treatment options depend upon woman wishes and her concerns about symptoms control or subfertility. If woman want symptoms control and infertility is not an issue than she should provide hormonal treatment. For symptoms control. Use of COCP, progestogens, danazole and GnRH agonist all are equally effective in reliving symptoms by inhibiting ovulation but with different side effects profile. Preoperative assessment of disease extent and when clinical evidence of deeply infiltrating endometriosis, ureteric, bladder and bowel involvement should be assessed. Consider MRI, TVS IVP and barium enema studies to map disease. Surgical option consider if no response to medical treatment. Practice is to diagnose and ablation of all lesions for pain relief by laproscopy. Laparoscopic uterine nerve ablation does not relieve pain by itself and associated with uterine prolapsed and urinary problems. There is inconclusive evidence of presacral neurectomy. No role of preoperative and postoperative hormonal treatments. There is no role of complementary therapies in relieving endometriosis pain. Treatment should individualize according to patient need and wishes. If woman concerns is infertility than both partners should be evaluated even in presence of obvious cause. She needs treatment as there is no role of expectant management in moderate to severe endometriosis with infertility. There is no evidence of surgical treatment in improving infertility, but aim of surgery is to correct pelvic anatomy  to improve ease of monitoring and easier access to ovarian follicle for aspiration .In view of this laproscopy should arranged for ablation of visible lesions and restore pelvic anatomy. Endometrioma more than 4cm should be excised instead of ablation. Tubal flushing appears to improve pregnancy rate. IVF is appropriate treatment and met analysis showed down regulation with GnRH analogue for 3 months before IVF can improve live birth rate.                                                                
ESSAY 346 Posted by Gulshan K.
  1. I will ask about severity of her symptoms and their effects on quality of life. I will ask about her menstrual history .Enquire about gastrointestinal symptoms like change in bowel habit, alternating constipation and diarrhea, rectal bleeding and weight loss will show gastrointestinal cause or its involvement in disease. Involvment of Urinary symptoms like bladder pain. Past history of PID and history of subfertility. Obstetrics history. Sexual abuse history. Enquire about psychological symptoms. Clinical examination of abdomen for tenderness, rebound tenderness, abdominal mass and distention. Pelvic examination to assess uterine size, mobility, its position, tenderness, adnexal tenderness, cervical tenderness and presence of nodules in   rectovaginal septum. Per rectal examination in case of rectal bleeding.
  2. Criteria include Signs and symptoms of endometriosis but they are nonspecific. Serum marker of CA 125 level do not provide adequate diagnostic accuracy.TVS is useful for diagnosis of endometrioma and normal finding may reassure woman  but it cannot diagnose peritoneal endometriosis. MRI may be useful in diagnosis of deeply infiltrating endometriosis. Diagnostic laparoscopy for visual inspection of pelvic organs and positive
Sofia Posted by sofia  S.

 

A healthy 35 year old woman is referred to the gynaecology clinic because of a 12 months history of pelvic pain, painful periods and pain during intercourse. Her symptoms have not responded to simple analgesia. (a) Discuss your clinical assessment [6 marks]. (b) She is found to have moderately severe endometriosis. Discuss the criteria for making this diagnosis [4 marks]. (c) Discuss and justify her treatment options [10 marks]

 Ans A: initial clinical assessment would include taking history regarding the severity of symptom and effect on quality of life. Associated symptoms like altered bowel habit  , blood in stool, urinary symptoms like frequency ,urgency and bladder pain hematuria, mood changes like anxiety irritability and depression. History of psychosexual abuse should be enquired sensitively. Menstrual history for LMP , menstrual regularity and flow and previous gynecological pathology. Contraceptive history and reproductive intensions . h/o  STI and PID, previous treatment inculding surgery.

Examination includes BMI and  abdominal exam  for  palpable mass. Perspeculum and vaginal exam for abnormal discharge ,endometriotic spots, uterine tenderness ,mobility,adnexal mass and nodularity in POD.

Investigations  include high vaginal swab although poor sensitivity in diagnosing chronic  PID. transvaginal scan for pelvic pathology(sensitivity poor in detecting peritoneal endometriosis).MRI +/- IVP/ Barium enema depending on clinical suspicion. Diagnostic laproscopy is gold standard  for diagnosing causes for pelvic pain but invasive and associated with risk of surgery and anesthesia.

Ans b:  Severity is graded  by simply describing the finding at surgery  and biopsy of lesion or quantitatively using American society of reproductive medicine classification system   with following components- superficial and deep endometriotic  lesion on ovaries and peritoneum ,obliteration of POD, and dense and flimsy adhesions on tubes and ovaries and a score of 16-40 is considered moderate and> 40 is considered severe endometriosis. No correlation between such system and type and severity of pain.

Ansc: Her treatment would be based on providing pain relief, preventing disease progression and addressing fertility issues if desirous of future pregnancy. Individualized   multidisciplinary management involving consultant gynecologist with expertise in advance laparoscopic surgery, specialist nurse, psychotherapist and pain  clinic specialist. Initial management involves pharmacotherapy . Simple analgesia like NSAIDS  provide some degree of pain relief but do not effect the disease process and prolonged treatment is limited by  gastric side effects. Hormonal therapy includes COCS  progesterone  GnRH agonist, danazol and LNG IUS. Suppression of ovarian function for 6 months  reduces endometriosis associated pain. All are equally effective  use is limited by  response to therapy and side effect profile.GNRH agonist associated with loss of BMD and requires addback therapy while danazol causes androgenic side effects. Symptom recurrence is common  after stopping medical treatment.

Surgical management indicated in case of failure of medical therapy. May involve bladder and bowel so input for surgical team would be required. Recommended surgery is laparoscopic ablation or excision of endometriotic implants with adhesiolysis. Excision is associated with lesser chances of recurrence than ablation. In order to completely excise  deep infiltrating lesion  hysterectomy with b/l salphingoopherectomy may be required in certain cases ( followed by HRT).Role of presacral neurectomy for pain relief  is uncertain. Role of pre or post op hormonal treatment for pain relief is not justified by evidence. Complementary therapy like homeopathy reflexology and acupuncture and TENS are not evidence based but should not be ruled out if women feels that its beneficial to her pain management and quality of life.

If desirous of fertility there is no role of medical management and role of surgery in improving pregnancy rate in moderate to severe endometriosis is uncertain. She would require IVF  and cystectomy is recommended for endometrioma > 4cm. GnRH agonist  for 3-6 mnths before IVF increases clinical pregnancy rate. Information regarding self help and support groups should be provided.

 

ANSWER TO ESSAY 346 Posted by DHARSHITHA J.

 

(a)I will take a detailed history inquiring other symptoms of endometriosis -such as menorrghagia, dyschasia, rectal bleeding and haematuria mid ovulatory pain , chronic fatigue and their relationship to her menstrual cycle

I will inquire regarding any symptoms relating to PID including history of previous pelvic infection, TOP(Termination of pregnancy), presence of vaginal discharge and history of recurrent pyrexia.

I will also assess the impact on her social and sexuel life due to this condition.

I will inquire regarding her bowel and urinary symptoms to exclude any relevance to IBS and possibility of interstitial cystitis.

I wiill also ask about any history of infertility and previous osbstertric history and future fertility scopes and her current contrcaeptive method.

 

On examination , I will assess her BMI as it will be helpful to determine the treatment options, also abdominal examination to exclude masses such as fibroids and ovarian cysts(endometriomas), also to assess any abdominal tenderness guarding or rigidity.

In addition I will perform a speculum examination to exclude any signs of cervicitis and vaginal discharge. At the same time I will take genital swabs including HVS, endocervical swabs and low vaginal swabs to exclude pelvic infection, including chlamydia and gonococcus.

Also I will perform pelvic examination to assess the uterine size ,direction and the mobility as in endometriosis the uterus can be fixed and retroverted. Also I will asseess cervical excitation adenexial tenderness, and induration and thickening of uterosacral ligaments. In addition I will exclude any adenexial masses that could be endometriomas.

 

As investigations I will perform FBC to exclude anaemia, and leucocytosis and CRP levels to exclude any on going infection (PID)and CA-125 levels( Helpful to identify patients who benefit by early treatment)

 

U/sscan will help to exclude the presence of endometriomas and any other pelvic pathalogy such as fibroids, but not a sensitive and precise investigation to diagnose endometriosis.

 

Diagnostic laparoscopy is the ideal goldstandard for the diagnosis of endometriosis, at the same tme the patency of the fallopian tubes can be asssessed, also mild-minimal endometriosis can be treated by abalation.

 

Cystoscopy and sigmoidoscopy can be performed if history suggestive of rectal bleeding or haematuria during menstruation to exclude rectal or vescical endometriosis

 

(b)The criteria depend on the laparoscopic findings of endometriosis ranked against the American Fertility Society Staging of endometriosis.

According to this staging procedure it assess the nature of the peritoneal, ovarian and tubal adhesions and the direct involvement of tubes and ovaries(presence of endometriomas). According to these findings a score is given.

Whether the adhesions are dense or filmsy, and the presence of endometriomas and their size, length of the tubes involves will determine the stage.

(c)The management options depend on patient,s future fertility scopes , degree of the symptoms and their impact on the lifestyle, willingness for medical/surgical management.

 

If she does not willing to retain her fertility or if her familly completed, aim would be the symptom controll and the first potion would be medical management. This comprises hormonal related and non hormonal treatments.

Non hormonal treatments are NSAIDS such as Mefenemic acid, but this can cause gastric irritation, but no effect on the fertility.

Hormonal treatments includes COCP, progestogens(Depoprovera).COCP helps good cycle controll and also a contraceptive and offers additional non contraceptive benefits. Depoprovera assosciates with progestogenic side effects such as weight gain, abdominal bloating breast tenderness, and also causes irregular bleeding pv.

Gestrinone is a 19-nortestesterone effective for endometriosis having androgenic antioestrogenic and anti progestpogenic effects, needs effective contraception to prevent virilization of the female fetus.

Danazole also effective , but more androgenic sideeffects such as acne , hirsuitism, clitoromegally and deepening of the voice, some,which are irreversible. This is generally given only for 6 months duration due to sifde effects and needs effective contraception to avoid masculinization of female fetus, but noeffect on the bone loss

GnRH analogues is effective but limited due to vasomotor symptoms and osteoporosis, needs addback therapy if given beyond 6 months, either with tibolone, oestrogen or progestogen.

Danazole and GnRH analogues not used as first line options due to their side effects.

However all above medications are efective against endometriosis, in equal capacity ,but the side efffect profile and individual response determine the appropriate therapy.

 

LNG-IUS may also an option and can be used for symptom controll.

 

Other steps involved are referring her to chronic pain management team/ clinic, referring to a psychologist and for counselling, and refer to other complementary therapies such as acupuncture, TENS, homeopathy, Hypnotherapy.

 

If medical or complementary treatments are not responding, surgical methods can be considered and offered.

LUNA(Laparoscopic uterine nerve ablation), Presacral neurectomy, pelvic clearence are the availaible options.

LUNA is easier to perform but the effectiveness is doubtful , however presacral neurectomy requires surgical expertise and assosciated with more morbidity , but shown to be effective than LUNA.

Pelvic clearence is considered if the woman has completed her familly and if other methods failed, and symptoms are severe or co-existing other gynaecological pathalogy such as fibroids present. HRT should be discussed prior to the procedure. Pre treatment with GnRH analogues may be helpful to identify women who benefit from this procedure. However this involves removal of uterus ,tubes and ovaries and other endometriotic patches and the procedure itself assosciated with higher morbidity.

 

If fertility is desired, and the pregnancy is a concern, IVF treatment is the best option for her as it is moderate to severe endometriosis and she is now 35 years.In this case, just laparoscopic endometrial ablation is not helpful (such as in mild to minimal endometriosis)

 

Prior to the IVF treatment, bilateral clipping of uterine tubes, cystectomy for endometrioma larger than 4cm, pretreatment with GnRh analogues will improve the success rates of IVF treatment.

 

H H Posted by H H.

Will take history including information about the pai,site,type eg colicky may be intestinal,radiation of pain,what increase and what decrease it and if she took medications for it. Will  ask of effect of pain on her quality of life.  Will ask of her LMP to see  if possibility of associated pregnancy. Will see if she has regular period, Will ask of her previous obstetric history and mode of delivery as if had cesarean  ,might have adhesions. Will ask of her fertility wishes . Will ask if using contraception ,type of contraception and of her sexual history, number of partners and any previous history of sexually transmited infections. In her medical history will ask of history of irritable bowel syndrome,inflamatory bowel disease, urinary tract infection and if she had rectal bleeding . Rectal bleeding during menstruation may point to endometriosis.Will see if there is hiden agenda sensitively eg domestic abuse or sexual abuse.Will ask if had previous surgery.

Will perform general examination for pallor,pulse, BP and BMI.Abdominal exam for tenderness,rigidity, rebound tenderness, abdominal distension, and abdominal masses. Vaginal examination for uterine size,position and fixity,. Cervical motion tenderness,adnexal masses, and nodular structures in Douglas pouch .Rectal examination for rectal bleeding.Vulvo vaginal fistulae in Crohn’s disease.

Investigations will include FBC for anemia,WBC and CRP will detect inflamatory cause.  Serum CA125 if found elevated may point to endometriosis. Transvaginal ultrasound  and MRI for adnexal masses .diagnostic laparoscopy which is the gold standard.

 

B) Patient with these symptoms in history can be due to endometriosis specialy if pain occurs before and during menstruation. Feeling of nodules on pevic examination specially if this is done during menstruation can point to endometriosis. CA125 is elevated in patients who have endometriosis. Actual diagnosis is by doing laparoscopy and seeing endometriotic patches .To confirm the diagnosis biopsy is taken and histo pathological examination done.

C) There is the option of expectant treatment (no treatment) . Medical options to releive pain include use of non steroidal anti inflamatory drugs but evidence does not support their effectiveness in endometriosis. Combined oral contraceptives can be used on a continuose or cyclical pattern to releife pain provided patient fullfil UKMEC 1 or 2. Progestogens as progesterone only pills, depot provera, nexplanon are effective in releife of endometriosis related pain. Patient should be warned of side effects as irregular vaginal bleeding, bloatedness ,weight gain and acne. Amenorrhea can occur specially with depot provera. Mirena coil can be used to releife pain ,acts locally so lowers the previous systemic side effects of progetogens ,however the evidence that it is effective against endometriosis related pain is weak.

Danazol is effective in pain due to endometriosis but this is limited by androgenic side effects ( deepning of voice, scalp hair recession, atrophy of breasts and virilisation), so not used more than 6 momths at doses of 100mg daily.

GNRH agonists are effective in releif of endometriosis related pain ,but there is the problem of severe menopausal symptoms and loss of bone density if used more than 6 months. Add back therapy with Tibolone can allow its use for longer times.

Surgical options include laparoscopic ablation or excision of endometriotic patches and division of adhesions. This requires surgical expertise and proper counselling and consent of patient regarding visceral and vascular of laparoscopy. There is no evidence that laparoscopic uterine nerve ablation LUNA is effective in treatment of pain due to endometriosis. Presacral neurectomy is effective but is more difficult.

In patients who completed their family,total andominal hystrectomy and bilateral salpingo oophrectomy is an option. HRT counselling  should be discussed before this step.

At the end maternal wishes should be respected for which treatment given. Acupuncture is not effective. Homeopathy can be used.

   

Posted by amr G.

According to the history given,the patient is suffering from chronic pelvic pain,and endometriosis is the most likely diagnosis.Assessment include history taking regarding onset, coarse and duration of the symptoms.The severity and character of pain( relation of the pain to the periods,aggrevation of pain with start of the period is suggestive of endometriosis) should be explored.Character of dysparunea, wether superficial or deep, sholud be asked about.The presence of bowel symptoms, like pain with defecation ( suggestive of endometriosis), or chronic constipation( suggestive of irritable bowel syndrome).Fertility and contraception is important to define the cause of pain and different treartment options.Abdominal examination can detect the presence of any abdominal masses (fibroid uterus, large endometiomas).Vaginal and bimanula examination can reveal the presence of fixed retroverted uterus in case of endometriosis , or adenexal masses.Speculum examination to detect signs of infection or cervical abnormalities.Investigations should include transvaginal ultrasound scan( toc check for uterine or ovarian pathology eg.endometrioma, fibroid).If no cause is found for the patient symptoms, laparoscopy to detect the presence of endometriosis should be performed.

Criteria to define the degree of endometriosis is  the extent of endometriosis , structures involved and presence (and extent) of pelvic adhesions.All these criteria can be defined by laparoscopy.Moderate to sever affection means affection of ovaries, douglas pouch, uterosacral and round ligamnets.These alsolaso include the presence of large sized lesions and dense pelvic adhesions.

There are many treatment options which depend on severity of endometriosis  desire for fertility and need for contraception.If the patient is desiring fertility, then, surgical treatment is indicated.Surgery includes ovarian cystectomy ( if large endometriomas are present).Laser or diathermy ablation of the endometriotic foci should be done.This can be achieved by laparoscopy or laparotomy.Tubal patency should be checked during laparoscopy and adhesiolysis may be indicated if tubal adhesions are detected.This should be followed bey intrauterine insemination with or without induction of ovulation, for six months.If failed, the, IVF can be offered.If the patient is not desiring fertility, hormonal treatment aiming at relief of pain should be adopted.Progestogens ,eg.norethistrone 15 mg/day , continously for at least three months can be offered.Combined oral contraceptive pills , either cyclic or continous, can be used for an average of 6 months.If symptoms are still not responding, the LHRH analogues can be offered for 4 months, however, vasomotor symptoms(hot flushes), and risk of osteoprosis should be accpeted by the patient, add back HRT , using tibolone, progestogens or low dose estrogen cane be offered to decrease the side effects of treatment.Intrauterine mirena system, can be used, al;though the chance of success will not be high with sever endometriosis.Aromataze enzyme inhibitor like Letrazol can be used also.Radical treatment , in the form of total abdominal hysterectomy + bilateral salpingoopherectomy can be offered after failure of all other options , if the patient has completed her family.Hormone replacement therapy should be offered and disscussed before surgery.Patient involvment in the treatment startegy is essential.

Posted by miss T.

(a)

I will give her dedicated time to speak freely about her ideas of pain in detail. I will then inquire about pattern of pain and menstrual history. Cyclic pain points to a gynecological cause. Associated urinary, bowel, musculoskeletal symptoms to rule out irritable bowel syndrome, interstitial cystitis, fibromyalgia and the effect of these symptoms have on her quality of life. I will make a sensitive inquiry about the history of sexual abuse in the past.

I will then do abdominal examination for tenderness and any organomegaly. Speculum examination for abnormal vaginal discharges. Bimanual examination for uterine size, mobility, tenderness and adnexal masses. It should preferably be done at the time of cycle as deep endometriotic nodules can be felt easily at that time.

Investigations will be done on the basis of history and examination findings. I will take STI screen if she is at high risk of sexually transmitted infection (multiple sexual partners, young age <25 years) If any abnormality is detected clinically then pelvic ultrasound +/- MRI can be carried out. USS is good in diagnosis of fibroid uterus, ovarian endometrioma, adnexal masses but adenomyosis and endometriotic nodules can be missed on ultrasound and MRI is needed. Laparoscopy is a second line investigation if empirical medical management has failed to relieve symptoms or if there is high index of suspicion of endometriosis. It has further advantage that tissue for biopsy can be taken at same time. But the disadvantages are that a laparoscopy may lead to diagnosis that is just a co-incidental finding and not the main cause of symptoms. Availability of equipment and expertise is also an issue.

(b)

Definitive diagnosis of endometriosis can be made on laparoscopy with histopathology. Findings of endometriosis are variable and can be a powder burn lesion, a blue spot, white plaque on the ovaries, serosal surfaces, and pouch of Douglas. The site and size of all lesions found at laparoscopy should be documented or ideally recorded.

The staging of disease is based on American fertility society classification system which takes account of appearance of lesions, deep infiltrating nodules, ovarian endometrioma, involvement of tubes ovaries and obliteration of the pouch of Douglas. Scoring is given for presence of these features and a final score is calculated which classify the disease as mild, moderate or severe. The classification system is good to stage the disease on visualization but it correlates poorly with the clinical symptoms.

(c)

She will be referred for multidisciplinary care where options of not only medical but also advanced surgical treatment should be available. Management plan will be made in partnership with her and the chronic nature of disease plus need of long term treatment will be made clear to her. Treatment is based on most presenting symptom, her fertility needs and motivation.

If fertility is not an issue and her main concern is relieve of pain then options for medical management are ovarian suppression with hormones reduces endometriosis associated pain but the pain return on discontinuing treatment. Options are combined oral contraceptive pills, medroxyprogesterone acetate, LNG-IUS, GnRH analogues, danazol and gestrinone. All are shown to reduce pain but the cost and side effect profile differ for all of these and hormonal preparation may not be preferred by her since suffering for 12 months she will be looking for a long lasting treatment.

Surgical treatment options are operative laparoscopy with ablation/resection of all visible lesions. It has the advantage of practically eliminating all visible disease so is more effective with long lasting results but it need surgical expertise and has known risks of anesthesia, bowel perforation, hospital admission. If there is bladder and bowel involvement with the disease, she will need an extensive surgery in a tertiary care setting. A laparotomy with resection of endometriosis with stoma formation or urinary diversion as needed and surgeons (colorectal/urologist) will need to involved. Pre or post surgical treatment with hormones is of no proven benefit.

Pelvic denervation may not be a good option for her as the evidence of benefit is limited but it may be considered if pain remains refractory to other treatment modalities. Laparoscopic uterine nerve ablation involves division of sensory sympathetic and parasympathetic supply to uterus at the uetrosacral ligament level. It has shown be of no long term benefit and has been associated with risk of uterine prolapse so I will not offer her with this. In laparoscopic pre-sacral neurectomy hypogasrtic nerves is divided at sacral promontory. It requires much higher level of surgical skill and has higher morbidity. It is proved to be better in pain control that surgical resection alone but the complexity of procedure makes it unavailable at most centers.

Total abdominal hysterectomy and bilateral salpingo-oophorectomy is the treatment of last resort. She will need to be on hormone replacement therapy afterwards.

If she wants to get pregnant she will be informed that surgical resection of endometriosis for improving fertility rate is of no proven benefit but if there is ovarian endometrioma, it will be removed surgically as it is shown be of benefit in improving pregnancy rates. Her best treatment option is in-vitro fertilization. If there are distorted dilated tubes, surgical bilateral salpingectomy is shown to improve success rates of IVF.

Psychological support and long term follow up is the mainstay of treatment. A friendly relation ship with doctor is necessary.

Hbadran Posted by HAnaa B.
A Assessment of the patient started from taking good history about pain score , radiation , factors relief and factors increase the pain, its relation to the start (dysmenorrheal)or the end of menstruation endometriosis ), presence of intermenstural bleed( PID ).mid cycle (pain ovulation pain), its relation to micturation (interstitial cystitis increased by bladder filling ),its relation to defecation in IBS specially if accompanied by bleeding and mucous. Pain relation to intercourse deep or superficial dysparunia, fertility issues should be asked also either she was able to get pregnancy or no, the way of delivery. Previous repeated abdominal surgeries.Qol life should be also assisted by special questionnaire. Examination for abdominal tenderness, mass, pelvic exam will appoint uterine tenderness adnexal mass or rectovaginal septum tenderness. Investigation of HVS, urine cytology, culture and stool occult blood, pelvic us, pelvic MRI and laparoscopy. DD includes chronic pelvic pain, PID, IBS, Ovarian cyst, pelvic endometriosis, uterine adenomyosis, pelvic adhesions, fibroids, muscular pain. B Diagnosis of sever endometriosis is mainly depending on laparoscopic findings after counseling and explaining the risk of the operation. Seeing lesions biopsy assessment of the peritoneum POD, ovaries and tubes according to special score system given by the American society of endometriosis score of 16-30 points is classified as type 3or sever endometriosis. MRI, is one of the non invasive modalities that diagnosis interstitial or infiltrating lesions. Level of CA125 more than 1000 may appoint to the sever disease non specific test. T/V US can diagnosis ovarian endometriomas.,But can’t appoint to the tubes and the peritoneum. C Treatment modalities available should be discussed with the women according to her wishes and expectations after failure of simple analgesia. Women not wishing to get pregnant and her main concern towards pain relief .options of continuous contraceptive piles for 3 months tricycle. Cost-effective and less side effect that other hormones. Medroxy progesterone acetate injection for 3 months is effective in pain relief but with menstrual irregularies. High doses of progesterone may be also effective but side effect (depression, bloating and hypoestrogenmia ). Danazol (antiestrogen ,antiprogesteron,) its effect is limited by wide side effect profile gesterinone is the same action with less side effect. Contraception should be used for both, because of virulizing side effect of both medications. GNRH agonist used for 6 month alone or for 2 years with add back estrogen and progesterone to counter effect bone loss of density. will bring regression of the endometrum and decrease pain . Mireana system can be used also for 3 years. She should always know that the pain is never completed, recurrence is common in 5 years . Surgical treatment can be also done during diagnosis with laparoscopy after consent. Removal of visible lesions or laser ablation of detected foci. Pelvic clearance should be done in severe cases with add of HRT after risk assessment. In case she is seeking to get pregnancy , no role for medical treatment, surgical removal or ablations followed by IVF after 3-6 month of GNRH agonist. Ovarian cyst more than 4 cm is to be removed before IVF. Adhesolysis alone effect is not certain on pregnancy rate.
hbadran Posted by HAnaa B.

A

Assessment of the patient started from taking good history about pain score , radiation , factors relief and factors increase the pain, its relation to the start  (dysmenorrheal)or the end of menstruation endometriosis ), presence of intermenstural bleed( PID ).mid cycle (pain ovulation pain), its relation to micturation (interstitial cystitis increased by bladder filling ),its relation to defecation in IBS specially if accompanied by bleeding and mucous. Pain relation to intercourse deep or superficial dysparunia, fertility issues should be asked also either she was able to get pregnancy or no, the way of delivery. Previous repeated abdominal surgeries.Qol life should be also assisted by special questionnaire.

Examination for abdominal tenderness, mass, pelvic exam will appoint uterine tenderness adnexal mass or rectovaginal septum tenderness.

Investigation of  HVS, urine   cytology, culture and stool occult blood, pelvic us, pelvic MRI and laparoscopy.

DD includes chronic pelvic pain, PID, IBS, Ovarian cyst, pelvic endometriosis, uterine adenomyosis, pelvic adhesions, fibroids, muscular pain.

B

Diagnosis of sever endometriosis is mainly depending on laparoscopic findings after counseling and explaining the risk of the operation. Seeing lesions biopsy  assessment of the peritoneum POD, ovaries  and tubes according to special score system given by the American society of endometriosis score of 16-30 points  is classified as type 3or sever endometriosis.

MRI, is one of the non invasive modalities that diagnosis interstitial or infiltrating lesions.

Level of CA125 more than 1000 may appoint to the sever disease non specific test.

T/V US can diagnosis ovarian endometriomas.,But can’t appoint to the tubes and the peritoneum.

C

Treatment modalities available should be discussed with the women according to her wishes and expectations  after failure of simple analgesia.

Women not wishing to get pregnant and her main concern towards pain relief .options of continuous contraceptive piles for 3 months tricycle. Cost-effective and less side effect that other hormones.

Medroxy progesterone acetate injection for 3 months is effective in pain relief but with menstrual irregularies.

High doses of progesterone may be also effective but side effect (depression, bloating and hypoestrogenmia ).

Danazol (antiestrogen ,antiprogesteron,) its effect is limited by wide side effect profile gesterinone is the same action with less side effect. Contraception should be used for both, because of virulizing side effect of both medications.

GNRH agonist used for 6 month alone or for 2 years with add back estrogen and progesterone  to counter effect bone loss of density. will  bring regression of the endometrum and decrease pain .

Mireana  system can be used also for 3 years.

She should always know that the pain is never completed, recurrence is common in 5 years .

Surgical treatment can be also done during diagnosis with laparoscopy after consent. Removal of visible lesions or laser ablation of detected  foci.

Pelvic clearance should be done in severe cases with add of HRT after risk assessment.

In case she is seeking to get pregnancy , no

role for medical treatment, surgical removal or ablations followed by  IVF  after 3-6 month of GNRH agonist.

Ovarian cyst more than 4 cm is to be removed before IVF.

Adhesolysis alone effect is not certain on pregnancy rate.

Essay 346 Posted by Reena G.

a)For clinical assessment I would like to ask about her pain, location, any radiation, nature of pain colicky or continuous, timing of pain ,its relation to menses,defaecation, micturitionor movement, severity of pain whether associated with nausea and vomiting. LMP., Cycle, menorrhagia, vaginal discharge ..Hx of recent change of partner, past hx of PID, any contraception use like IUCD, Hx of sexual abuse to be asked in sensitive manner, psychosocial hx about occupation and stress, decrease in weight, GI symptoms(constipation ,diarrhoea, any change in stool form , passage of mucus or stool), GU symptoms( dysuria, frequency). In examination per abdomen to know any mass, tenderness, rebound tenderness ,rigidity, VE to know uterine size , fixity or mobile, any cx tenderness, any adnexal tenderness, per rectal examination needed if rectal bleeding.I would do certain investigation to reach the diagnosis like FBC and CRP (tell about inflammatory process), uretheral , endocervical swab for culture and Chlamydia  ELISA/PCR.MSU- dipstix for blood, microscopy and cultire TVS for uterine and ovarian pathology, will tell about ovarian endometrioma but not peritoneal endometriosis.Diagnostic laparoscopy is the gold standart  will detect peritoneal endometroisis  and adhesions  and also help in taking biopsies from suspected lesions.MRI will tell about peritoneal endometrioses and adenomyosis.

b)The classification of endometrioses is subjective and poorly correlates with pain according to American Fertility society Scoring system  the scores are given  according to size of lesion and its depth of involvement of peritoneum, B/L ovaries and B/L tubes. So the criteria for severity is given according to scores. Minimal 1-5, mild 6-15, Moderate 16-30, Severe 31-54 scores. For peritoneum size of lesion <1cm (1), 1-3(2), >3cm (3), peritoneal adhesions film (1), dense with partial obstruction of POD (2), complete obstruction (3). For Ovary Lesions <1cm (2), 1-3cm (4), >3cm  or ruptured endometrioma(6), ov. Adhesions  Filmy, partial enclosure and complete enclosure given 2,4,6 scores, score for each ovary . For tube ,1cm, >1cm and tube occluded assigned 2,4,6 score and tubal adhesion filmy, dense with distorted tube and complete occluded tube given 2,4,6 score for each tube separately.

c)I will dicuss with her in detail about course of disease and various options and its potential side effects that will enable her to make informed decision about  most appropriate modality, will know her principal concerns, her pain, infertility, age or contraception issues. COCP is the first line of management , help in relieving pain effectively and provide contraception can be given cyclical, tricyclical or continous, Progestogens in adequate dose like DMPA provides effective relief in pain and amenorrhoea but can have irregular bleeding, weight gain , mood change, depression, acne, bloating and prolonged use may lead to hypooestrogenimea and loss of bone mineral density . Danazol is androgenic, antio-estrogenic , anti progestogenic drug , has direct inhibitory effect on endometrium , increase libido not associated with decrease in BMD, its effective but use is limited due to side effects acne, hirsuitism,greasy skin, weight gain , hot flushes and breast atrophy which is reversible but deepening of voice is irreversible. Women shoul use effective contraception to prevent virilization of female fetus.Getrinone is 19 nortestosterone anti-oestrogen and anti-progestogen administered twice weekly has mild androgenic side effects need adequate contraception during its use.GnRH agonist  is effective  but its use should be limited to 6 months as associated with menopausal side effects, 6% loss of BMD if use more than 6 months, add back therapy of tibolone, a progestogen  or low dose oestrogen  may allow GNRH to be used till 2 years. LNG IUS is effective in reducing pain and effect is maintained for 3 yearsSurgical treatment aim is to restore the normal anatomy, to excise the disease and to preserve the normal tissue. Laparoscopic ablation of moderate endometrioses is effective in relieving pain at 6 month follow up, excisional surgery is associated with significant improvement at 6 month.But effect of excision of moderate to severe endometrioses with adhesiolyses on pregnancy rate remains uncertain LUNA on its own not effective in relieving pain , pre sacral neurectomy is effective but is associated with morbidity. If  she wishes to conceive then IVF and ET is the choice  in her condition . Laparoscopic ovarian cystectomy is recommended for endometrioma > 4cm prior to IVF but risks of bowel and vascular injuries to be discussed.I will provide written information and will tell her about support groups like National endometriosis society.

answer essay 346 Posted by Sherif N.

1) clinical assessment

proper history taking about the presence of any vaginal discharge, history of previous PID or STIs, history of dysuria, burning micturition suggestive UTI, history of rectal symproms as constipation or symptoms of spastic colon. history of abdominal enlargement or any abdominal masses felt indicating fibroid uterus or ovarian masses.

menstrual history regarding the regularity, presence of intermenstrual bleeding, pattern of pain whether it is releived or increased by starting menstrual flow. presence of bleeding per rectum, or bleeding from any surgical scar in addition to any bluish discoloration

history of intake of COCP, progestagen, and their effect on the pain she is feeling

examination for any abdominal masses, PV for uterine mobility , presence of nodules in Douglas Pouch, Bandle sign and feeling the uterosacral ligaments, any ovarian masses felt suggesting endometriotic chocolate cysts

investigation in the form of high vaginal swab in the presence of vaginal discharge, midstream urine analysis for infection, CA125, US to detect ovarian masses suggesting endometriotic cysts

b)criteria for making diagnoisis:

endometriosis is more common in white population, with high sociaoeconomic classes, low or no parity, (so if the patient falls in this criteria she is more prone to endometriosis)

history of the triad of pelvic pain, dysmenorrhea, and dyspareunia

examination of fixed retroverted uterus, presence of nodules in Douglas Pouch, cord l;ike structure of uterosacral ligaments

raised CA 125

US revealing picture of chocolate cysts in one or both ovaries

c) discuss and justify ttt options:

patient is having moderately severe endometriosis which was not responding to simple analgesics, so COCP may be tried given regularly for 21days with 1 week free lasting for 6months to detect the response

MPA injections given every 3months with referral of its side effects from bloatedness, irritability

danazol to be given continuously for 6months and inform patient about its androgenic side effects

GnRH analogues, for 6months, but they may cause decreased bone density, so they are used for 6months, then stop to allow restoration of bone density, also add-back regimen may be used with them

if the medical ttt fail to releive symptoms in this patient with moderately severe endometriosis, then surgical ttt should be taken, laparoscopy and laser ablation of endometriotic foci, with adhesiolysis of any possible adhesions causing fixed retroverted uterus, also ovarian cystectomy in case of chocolate cysts.

patient should be informed about risk of bowel damage during surgery due to adhesions, also the possibility of recurrence in case of presence of microscopic foci which will grow up later

she is also adviced to get pregnant (if willing, and regnancy isn't contraindicated) as this will help of atrophy of endometriotic foci

follow up after surgery is required for reassurance and support amd to detect any recurrence

anuradha n Posted by anuradha N.

a)My clinical assessment includes history,examination,investigations

Detailed history regarding severity of pain,effect on her daily activities,aggrevating and relieving factors,h/o fever,detailed mentrual history,dysmenorrhea,amount of bleeding(clots),h/o infertility or subfertility,obstetric history-number of pregnancies/parity.Presence of pain or bleeding associated with change in bowel or bladder habits(rectal bleeding or hematuria).Past history of treatment for PID,sexual history to be taken,use of contraceptives,medications used.

Examination-General condition,BP,BMI,pallor

P/A-tendernessand massess.   P/S-High vaginal ,endocervical swabs,presence of vaginal endometriotic spots

P/V-Uterine size,position,mobility,tenderness,adnexal massess,tenderness,detection of nodules in POD

P/R if rectal bleeding is present. Examination is more reliable to diagnose endometriosis if done during menstruation.

Investigations-Hb%,FBC,CRP raised in PID ,

Transvaginal scan(TVS)-detects ovarian endometriomas,adnexalmass(TVS cannot detect peritoneal endometriosis)

Diagnostic lapscopy-is the gold standard to know the disease extent of endometriosis ,helps to take biopsy and rule out other causes 

MRI-Can be an useful non invasive tool helps to diagnose adenomyosis/deep endometriosis

b)The severity of endometriosis is assessed by laparoscopy and scored as per American fertility Society. The severity correlates poorly with pain but useful for infertility management and prognosis.The score is based on the size of the lesion(1-3),severity of adhesions over the peritoneum,POD,ovary,tube.Dense adhesions over the ovary,tube,distorted tube/occluded tube carry a score of 4-6.Endometriotic cyst of>3cm,ruptured endometrioma score of 6 .with above total score added and severity is graded as

Stage 1-mild 1-5,stage 2 -moderate 6-15,stage 3 severe -16-30,stage 4 -extensive 31-54

c)Patient needs to be councelled regarding the nature,course of disease,treatment options and side effects.This should be supplemented with written information and support groups such as national endometriosis society.Enable the woman to make informed decision regarding issuses of pain,subfertility,severity,age factor

If her concern is pain,then options for her are medical methods,laparoscopy,excisional surgeries.

Medical   methods-COCPs-relieves pain,cost effective,better if used continuosly/ tricycling,suitable if not trying to conceive.

Progestogens depot provera-effective drug but sideeffects are weight gain,acne,bloatig,loss of bone density on long term.

Danazole-effective but side effects limits its use,acne hirsutism weight gain,breast atrophy,greasy skin and hair,risk of benign hepatic adenoma.

Gestrinone-similar androgenic side effects like danazole milder form.

GnRh agonists-are effective , should be limited to 6 months as there is  6% bone loss after 6 months.If used for longer term addback therpy to be used.all of the above treatment are equally effective with different side effect profile.None of the above medications interfere with biological mec hanism.

LNG ius-Effective in reducing the pain upto 3 years.

Surgical treatment-Surgery (lapscoppy/laparotomy)is the main stay of treatment.In moderate to severe disease (deeply infiltratig endometriosis).Ureteric involvement (IVP) bladder,bowel (barium enema studies) ,MRI help to map the disease extent.She needs to be reffered to a centre where expertise to offer in a multidisciplinary context (advanced lapscopy or laparotomy).Excisional surgery is better than ablative techniques for relief of pain.Pre and post operative hormonal treatment doesnot have effect on symptom recurrence,pain relief.However LNG IUS has shown benificial effect in preventing recurrence in severe disease.Pelvic clearence(TAS+BSO)is considered with severe symptoms ,failed medical/conservative surgical treatment if fertility is not an issue.HRT to be considered with relative risks and benefits of estrogens and progesterones with recurrence.

Laparoscopic uterine nerve ablation(LUNA) on its own doesnot relieve endometriosis related pain, has long term risk of uterine prolapse and blader dysfunction.Pre sacral nuerectomy is more complex operation is more effective but greater morbidity than LUNA.

If fertility is an issue-there is no role of medical therapy.Laparoscopic excision adhesiolysis in moderate to severe disease on pregnancy rate is uncertain.Laparoscopic ovarian cystectomy for endometrioma of.4cm is better than ablation / coagulation.Post operative hormonal therapy has no effect on pregnancy rate.In severre disease tubes are distorted hence IVF is the effective treatment.Better results are obtained when ovarian endometrioma .>/ 4cm are excised.Treatment with GnRh analogues for 3-6 months are used before starting IVF.No evidence that ovarian hyperstimulation and IVF affect the disease progression.

 

Posted by zaitouni L.

a) Assessment :

I should take hitory including charachter of pain ,wheather pain is intermittent or continouse .History of associated chronic pain as headache or backache . I will ask about effect of pain on her quality of life . Associated diarrhea or vomiting to exclude irritable bowel syndrome . Associated dysurea ,frequency and suprapubic pain to exclude interstitial cystities .

I will ask about use of intrauterine device .History of pelvic inflamatory disease . I will ask sensitivel about sexual hitory  and hitory of sexual abuse. Prevouse treatment and investigations and outcome. Also ask about symtos suggetive of sever disease as postcoital bleeding  or rectal bleeding. Effect of posture or movement on pain

I will palpate the abdomen for tenderness or masses . Iwill do speculum examination and take swabes  . I will do bimaual examination to detect adnexal masses or tenderness  or tender cx motion .  or fixed retroverted uterus.

Posted by C M.

 

   a) Discuss your clinical assessment [6 marks].   

initial assesment would firstly meeting with the woman discussing her symptoms, her fears and thoughts about the symptoms may be and the symptoms implications on her health. I would take a thorough history including the nature of pain (constant, colicky, sharp), whether its only with her periods or is it at other times, whether its cyclical in nature and if there is any correlation to activity, movement or diet (any food that bring the pain on). I would also want to know about previous operations (possibility of adhesions) and if she has taken any hormonal therapy as yet ie COC. I would also enquire about urinary symtoms and if she has had any vaginal discharge. I would then performa an examination noting her temperature, heart rate - excluding signs of infection. An abdominal palpation would highlight any masses (ie ovarian cysts or fibroid uterus) and elicit any tenderness. A speculum would indicate vaginal discharge and a bimanual examination would be useflu to palpate any adnexal masses, palpable nodules, uterine tenderness or cervical excitation. I would also perform a high vaginal swab if any discharge was visible.

(b) She is found to have moderately severe endometriosis. Discuss the criteria for making this diagnosis [4 marks].

The diagnosis is made in part by history and also by Laparoscopy which is deemed as the gold standard in making this diagnosis. Ideally the laparoscopy should be performed by someone who is trained to diagnose and treat the endometriosis at the same time. The college recommends that a grasper be used in the seconday port inorder to move pelvic organs and plapatefor any nodularity. It is best performed under video so this can be reviewed at a later date.The findings should be recorded in the notes with accuaret diagram of findings.  Biopsies are considered helpful. As histological evidence is of use and if an endometrioma is present this can be removed at the same time. TV ultrasound is considered useful for diagnosising endometriomas alone and its use in invasive disease is limited

(c) Discuss and justify her treatment options [10 marks]

Her treatment options incluude medical hormonal and surgical. The combined oral contraceptive pill COCP is the first option that could be considered. It can be used long term, can reduce the symptoms of endometriosis as it helps minise the flow during the period suppresses ovulation all of which contribute to her symptoms. It has the added advantage in that it can be used long term as it is considered safe however shoe would need to be counselled about the risks of the COCP namale VTE risks and would thus require a heigh and weight and BMI as well as a BP perfomed initially after 3 months and then 6 monthly thereafter.

The second potential option for her is the progesterone only preparation which would be adviseable if she had had any contraindications to taking the COCP (ie VTE risk, age limitation, migraines). The POP can suppress ovulation as well as reduce premenstrual symptoms as well and it has been shown to reduce the symptoms of Endometriosis. This can be taken long term as well and is safe to do so however she would need to be counselled about the risks of irregular bleeding and its side effects namely weight gain, breast tenderness.

The IUS is also an option and this has been shown to suppress the symptoms of endometriosis and has that added advantages that she would not require regular monitoring as in the case of COCP and POP whilst actively managing her symptoms and in some cases can case a light discharge to amenorrheic state, hence can also act as an effective contraception. It however is invasive which has a risk of perforation of the uterus on insertion and also a risk that it gets expelled.

GNRH agonists can also be considered which are also deemed to work well unfortunately they are restrived in their use secondary to their side effect profile, (a 6% bone density loss with a 6 month cycle usage). Some people advocate for ad-back hormonal therapy whist these are being given which can be protective as far as bone density is concerned. GNRH agonists are therefore useful only in trying to decide if surgery will be of benefit

Surgial options are either TAH +/- BSO though this carried significant morbidity. It however would eliminate her symptoms, she may however require HRT as she is still youngA healthy 35 year old woman is referred to the gynaecology clinic because of a 12 months history of pelvic pain, painful periods and pain during intercourse. Her symptoms have not responded to simple analgesia.     

 

Essay Posted by Samira  K.

1-Obstetric History should be taken for eg.her parity ,any history of infertility..Her menstrual history duration and flow of blood.if it is heavy bleeding.History of PMS esp.mastalgia and headaches.history of Ovulotion pain.Any history of  abnormal vaginal discharges.Previous history of ectopic .History of contraception.Sexual history .Past history of acquired STD.Any history of intermenstrual bleeding.Among Non gynaecological causes any change in bowl habits .defecation with pain .or bleeding per rectal.

Examination during mensturation after consent of patient.Lower abdominal tenderness.any masses palpated abdominally.Speculum examination for sign of cervicitis,any purulent vaginal discharges from Cervix.Culture chlamydia and gonorrhea taken from endocervix.Do pap smear if not done recently.Bimanual examination for Uterine size,mobility,position,any nodules felt in POD.Cervical motion tenderness,adnexal tenderness or any mass felt in adnexa.Rectal examination if rectal bleeding

We will ask for cbc for WBC count.Depending on examination if uterine size is felt to be 12 wks.or any masses felt in adnexa we will ask for Transvaginal Scan and MRI to view deeply infiltrating endometrisis in pelvis.Laproscopy is the gold standard for diagnosis.We will assess her Impact of this pain on quality of life.

2-Diagnosis of severity can be made by laproscopy only.According to American fertility society Scoring system Stage 2 or moderate (score from 6-15)Stage 3 Severe endometriosis(score 16-30).based on size of peritoneal implants,Adhesions filmy or thick.Size of endometrioma in ovary.Ovarian adhesions if they occupy the whole ovaries..Each ovary should be scored separately.Tubal implants size and tubal occlusion.Tubal adhesions and if tube is distorted.Score each tube separately

Pain is not correlating with stage of endometriosis.

3-Her treatment options depend on her desire of fertility .Pain or fertility.Severity of pain ,Location of endometriosis

As she already tried simple analgesics we can offer her COCP (continous or cyclical) which is cheap ,effective retain her fertility,better side effect profile.

Progestogens in high doses is effective but might not be tolerated because of side effects like headche,bloating,Acne,Wt.gain,mood change & loss of BMD with prolong use.With LNG IUS & depoprovera there is irregular bleeding also.Effect of Mirena is maintained for 3 years.

Danazol,gestrinone are both effective but their use is limited by sideeffect profile Like Acne,hirsutism,breast atrophy,deepening of voice which might be irreversible.gestrinone has better sudeeffect profile.Both can virilise female fetus so use contraception.Mean time to recurrence of symptoms is 6 months .

GnRH analogue is effective but as it cause reduce BMD so use is limited for 6 months only.We can give it with add back HRT like tibilone.This will protect the bone.Mean time to recurrence of symptoms are 5 months.

Surgery can be offered like laproscopic ablation of endometriotic lesions.Excision of endometrioma if any.Excitional surgery and adhesiolys reduced pain significantly at 1 year of follow up & esp.for women want to converse their fertility.but the effect on fertility is uncertain.Insertion of Mirena after laproscopic surgery is effective in relieving pain.Excisional surgery is complex and related with bowl and bladder injury.tubal flushing with oil media increase her chance of fertility

IVF with embryo tranfer is an option for her if she desire fertility.Laproscopic ovarian cystectomy for endometrioma more than 4cm and GnRh analogue 3 months prior to procedure increses her chance of fertility.Outcome is contraversial.Some studies shows low pregnancy rates.

LUNA is not effective but Presacral neurectomy is effective for pain but is complex procedure related with morbidity

 

Endometriosis essay Posted by Dr Dyslexia V.

X

a)      History should include the nature of the pain, cyclicity and it’s relation to menses, relieving and exacerbating factors which could differentiate from gynae to non gynaecological causes. The effect on quality of life is important as it causes sick leave and interpersonal sexual relationship problems. Obstetrics history in regards to previous children or pregnancy loss and fertility interest could also help in diagnosis and management. History of pelvic inflammatory disease or sexually transmitted disease or presence of any foul smelling discharge could attribute to chronic PID. Other non gynaecological causes should be ruled out as well by asking history of any frequent bowel emptying, bloody or mucous faeces or dyschezia should be taken. Examination should include body mass index, blood pressure for choice of medication such as use of combined oral contraceptive pill (OCP) which could be not used. Abdominal examination to assess for any tenderness on palpation or presence of abdominal mass in uterine fibroid, endometrioma or adenomyosis. Pelvic examination to assess the vaginal for any endometriotic nodules or presence of any foul smelling discharge. Bimanual palpation of the uterus to assess for size and presence of tenderness in adenomyosis. Adnexal tenderness with fixed retroverted uterus with nodular pouch of doughlas could indicate endometriosis.

b)      Gold standard diagnosis of endometriosis are based on direct visualization of the pelvis and abdomen via laporoscopy  by an experienced gynaecologist. The severity is based on scorring by the modified American Fertility Association score. It is based on the extent of the lesion either superficial or deep and its distribution on the pelvis. It is also associated with the presence of adhesion of the endmetriosis either flimsy or densed adhesion. The presence of obliteration of the culdesac of pouch of douglas or presence of lesion on the uterosacral ligamen. And if there is presence of endometrioma. A biopsy should be taken especially in a lesion > 3 cm to rule out malignancy.

c)       She could use  NSAID’s with mild opiods such as tramadol, or the latest cox inhibitors for pain relief. The use of NSAID’s could cause gastrointestinal disturbances and gastritis with chronic use while chronic use of opiods might create dependence. COCP could be used as well and it could be used cyclicity or continuously for 3 cycles with withdrawal bleed in between. The use of this has added advantage of contraception but it might be not used if there is other contraindication such as obesity or smoking with the patient aged more than 35. Progestogens such as medroxyprogesterone acetate, norethisterone, dydrogesterone could be used. It could be used  continuously to  cease menses and endometrial activities. Its side effects include bloatedness, water retention, breast pain and mood disorders. The other drugs are Danazol and androgenic, antiprogesteron and anti estrogen. It’s side effects include virilization such as vocal cord thickening, hirsutism, acne and potential virilization of a female fetus as it is not a contraceptive. Gestrinone is a progestogen with androgenic activity and anti estrogen which is conveniently with twice a week dosing. It has similar side effect profile as Danazol. The other drug which are commonly used is the GnRH analogue. It’s initial use causes a 14 day flare up but then it resolves. It is then preferably started an early cycle of menses. It creates a menopausal like state thus the side effect of climatric type symptoms such flushing, palpitation, vaginal dryness and loss of bone mineral density which are reversible. The use of add back therapy using either estradiol or tibolone is used to reduce the side effect. The most commonest method used which is beneficial to patient who are keen for fertility is laparoscopy surgery. Laparoscopy could be used to either ablate or excise the lesion with no difference in the future recurrence. Adhesiolysis and restoration of the pelvic anatomy  is paramount in these surgeries. Endometrioma should undergo cytectomy by removal chocolate content and ablation of the cyst wall. The chance of recurrence of endometrioma is less with excision of the endometriotic cyst wall. And finally if the patient is not keen for fertility or completed her family then hysterectomy and bilateral salphingoopherectomy could be done.

Posted by Murad A.

A healthy 35 year old woman is referred to the gynaecology clinic because of a 12 months history of pelvic pain, painful periods and pain during intercourse. Her symptoms have not responded to simple analgesia. (a) Discuss your clinical assessment [6 marks]. (b) She is found to have moderately severe endometriosis. Discuss the criteria for making this diagnosis [4 marks]. (c) Discuss and justify her treatment options [10 marks]

 I will take a  History which include an assessment of severity of symptoms and associated GI / GU or other systemic symptoms and thier imact on overall quality of life , I will explore her Medical and surgical history and her  Menestrual history including regularity, amount and abnormal bleeding or discharge. Obstetrical history, fertility status and future child bearing plans . The sexual history , numbers of partners and use of hormonal contaceptives. I will do a physical exam , including her general health , BMI for " possible fittnes for surgey vs hormonal therapy ,, Abdominal examination to role out pelvic masses and tenderness ,, pelvic exam should include bimanual exam to assess mobility and excitation / tenderness of uterus or adnexia and R/O pelvic massess or nodularities .  The patients concerns and fears need to be explored .  

    The diagnosis of endometriosis can be suspected by relevant history and exam , nevertheless the diagnosis need to be confirmed either visually via a laporoscope or histlogicaly by biopsy . The use of laparoscopy has been considered as a gold standard for evaluation of chronic pelvic pains and assessment of pelvic organs , If significant endometriosis is obvious with various lesions of differrent ages " smoke ashes burns to  web combs " , then the diagnosis can be firmly declared , otherwise a biopsy of suspected lesions need to be taken , " the presence of gland and stroma confirms the diagnosis " .

    Treatment include conservative approach with explanation and reassurance . It can be enough for patient that understand the nature of disease and accept increasing the amount and adjust thier simple analgesia accordingly , Patients who are willing for should be advised to get pregnant as pregnancy itself will put thier symptomes in remission and delay progression of their disease .

    Medical treatment specifically Hormonal therapy is effective in eleviating the patients symptomes but all available options lack long term control and relaps is the usual after stopping treatments, efficacy is comparable but they differ much according to thier side effect profiles . Options include GnRH analouges which supress ovarian function ans cause medical menopause ,, side effects include the whole spectrum of menopause and its symptoms . Add back therapy has been shown to decrease the symptoms without decreasing the efficacy . Danazol " testosterone derivative " is highly effictive nevertheless it is being withdrawn from the hormonal arsenal because of it serious and significant virilizing side effects . The use of combined oral contraceptives " continuously or cyclical is effective but risks / benifits profile need to be cuatiously balanced , the advantage is that it can be used for prolonged periods with readily and quickly reversable effects under patient control .

 Progestoges " Oral  /  Injectable /  IUS" can be used with a comparative better side effects profile and a bit less efficacy

Surgical management include laparoscopy and excision/ ablation of endometriotic spots and adhesiolysis ,,, Definite treatment of endometriosis is castration with  Bilateral oopherectomy , The use of HRT later might reactivate some endometriotic spots and adhesions might even cause pelvic pains after the procedure . Hysterectomy is not necessay . carful counselling should be undertaken before embarking into this radical approach

    

by NA Posted by naila A.

 

Healthy 35 year old woman is referred to the gynaecology clinic because of a 12 months history of pelvic pain, painful periods and pain during intercourse. Her symptoms have not responded to simple analgesia. (a) Discuss your clinical assessment [6 marks]. (b) She is found to have moderately severe endometriosis. Discuss the criteria for making this diagnosis [4 marks]. (c) Discuss and justify her treatment options [10 marks]

  1. I’ll take a detailed history of her symptoms to know the site and the type of pain, relationship with menstrual cycle, aggravating and relieving factors and radiation to any other site. Details about dysmenorrhea should be asked, if it is relieved with the onset of menstruation or aggravated. Dysparunea is superficial or deep should be asked. History of any rectal or urinary bleeding with the onset of menstruation, is needed to assess the extent of endometriosis if it is the cause of her symptoms. Details of her obstetric history are obtained to know the number of children their mode of deliveries and number of miscarriages or terminations or ectopic pregnancies. Her aspirations for future fertility are determined. History of any sexually transmitted disease and treatment, number of sexual partners and any change in partner in past 6 months is required to assess her risk of PID. I’ll check the results of her last pap smear and inquire her for any abnormal vaginal bleeding or vaginal discharge. I’ll examine her to check her temperature, pulse, BP and BMI. I’ll palpate her abdomen for any tenderness or mass in pelvis. I’ll do speculum examination to check the state of cervix and take swabs from vagina and cervix for vaginal smear and culture and sensitivity. I’ll do vaginal examination to assess the size and mobility of uterus and any mass in pelvis. To assess any nodularity in pouch of douglas examination is best performed during menstruation but it is not comfortable for patient and has low acceptability.
  2. Gold standard for diagnosis is visual inspection at laparoscopy. Severity of endometriosis is classified on the basis of extent of disease seen at laparoscopy. Few superficial lesions are classified as type one disease. Some deeply infiltrating lesions with some adhesions is classified as type two and extensive adhesions or deeply infiltrating lesions or endometrioms is classified as type 3 disease. Clinical diagnosis is on the basis of history and examination. If signs  and symptoms  are suggestive of diagnosis such as triad of dysmenorrhea, dysparnunea and infertility, or  fixed retroverted uterus with nodules in pouch of douglas or palpable mass in fornix empirical treatment can be started which can indirectly prove the diagnosis if there is improvement. 
  3. Her treatment options depend upon her desire of relieve of symptoms alone or desire of fertility as well. If fertility compromise is acceptable to her for the relief of symptoms the treatment options are medical and surgical. medical treatment is hormonal and non hormonal. NSAIDS are used to relieve pain. RCTs have not suggested the use of NSAIDS as effective treatment. COCPs are used as first line medical treatment, they can be used as conventionally or continuously or tricycling.They are effective in symptom control and have contraceptive and non contraceptive benefits. Progestogens, medroxy progesterone acetate, norethisteron or dydrogesteronein sufficient doses to induce amenorrhoea are effective treatment but associated with weight gain, bloatedness , mood change/depression and mastalgia.Also there is risk of irregular bleeding.LNG IUS can decrease the load of disease in pouch of douglas and cause decrease in endometriosis associated pain.There is risk of irregular vaginal bleeding with its use.Danazol is androgenic, anti estrogenic and anti progestogenic , can cause relief of symptoms but is associated with virilization. Symptom recurrence is reported in 6.2 months. GNRH analogues are effective in relief of symptoms but associated with loss of BMD if used more than 6 months.The use can be prolonged to 2 yrs with add back therapy. Symptom recurrence is reported in 5.2 months. Surgical options include laparoscopic laser ablation of superficial lesions and excision of deeply infiltrating lesions and endometrioms.Laparoscpic uterine nerve ablation at its own is not effective in relief of symptoms. Pelvic clearance surgery is offered in severe cases where relief is not obtained with medical or conservative surgical treatment .HRT need to be prescribed post operatively.